Save 10% on All AnalystPrep 2024 Study Packages with Coupon Code BLOG10 .

  • Payment Plans
  • Product List
  • Partnerships

AnalystPrep

  • Try Free Trial
  • Study Packages
  • Levels I, II & III Lifetime Package
  • Video Lessons
  • Study Notes
  • Practice Questions
  • Levels II & III Lifetime Package
  • About the Exam
  • About your Instructor
  • Part I Study Packages
  • Parts I & II Packages
  • Part I & Part II Lifetime Package
  • Part II Study Packages
  • Exams P & FM Lifetime Package
  • Quantitative Questions
  • Verbal Questions
  • Data Insight Questions
  • Live Tutoring
  • About your Instructors
  • EA Practice Questions
  • Data Sufficiency Questions
  • Integrated Reasoning Questions

Hypothesis Tests and Confidence Intervals in Multiple Regression

Hypothesis Tests and Confidence Intervals in Multiple Regression

After completing this reading you should be able to:

  • Construct, apply, and interpret hypothesis tests and confidence intervals for a single coefficient in a multiple regression.
  • Construct, apply, and interpret joint hypothesis tests and confidence intervals for multiple coefficients in a multiple regression.
  • Interpret the \(F\)-statistic.
  • Interpret tests of a single restriction involving multiple coefficients.
  • Interpret confidence sets for multiple coefficients.
  • Identify examples of omitted variable bias in multiple regressions.
  • Interpret the \({ R }^{ 2 }\) and adjusted \({ R }^{ 2 }\) in a multiple regression.

Hypothesis Tests and Confidence Intervals for a Single Coefficient

This section is about the calculation of the standard error, hypotheses testing, and confidence interval construction for a single regression in a multiple regression equation.

Introduction

In a previous chapter, we looked at simple linear regression where we deal with just one regressor (independent variable). The response (dependent variable) is assumed to be affected by just one independent variable.  M ultiple regression, on the other hand ,  simultaneously considers the influence of multiple explanatory variables on a response variable Y. We may want to establish the confidence interval of one of the independent variables. We may want to evaluate whether any particular independent variable has a significant effect on the dependent variable. Finally, We may also want to establish whether the independent variables as a group have a significant effect on the dependent variable. In this chapter, we delve into ways all this can be achieved.

Hypothesis Tests for a single coefficient

Suppose that we are testing the hypothesis that the true coefficient \({ \beta }_{ j }\) on the \(j\)th regressor takes on some specific value \({ \beta }_{ j,0 }\). Let the alternative hypothesis be two-sided. Therefore, the following is the mathematical expression of the two hypotheses:

$$ { H }_{ 0 }:{ \beta }_{ j }={ \beta }_{ j,0 }\quad vs.\quad { H }_{ 1 }:{ \beta }_{ j }\neq { \beta }_{ j,0 } $$

This expression represents the two-sided alternative. The following are the steps to follow while testing the null hypothesis:

  • Computing the coefficient’s standard error.

hypothesis test for multiple linear regression

$$ p-value=2\Phi \left( -|{ t }^{ act }| \right) $$

  • Also, the \(t\)-statistic can be compared to the critical value corresponding to the significance level that is desired for the test.

Confidence Intervals for a Single Coefficient

The confidence interval for a regression coefficient in multiple regression is calculated and interpreted the same way as it is in simple linear regression. 

hypothesis test for multiple linear regression

The t-statistic has n – k – 1 degrees of freedom where k = number of independents

Supposing that an interval contains the true value of \({ \beta }_{ j }\) with a probability of 95%. This is simply the 95% two-sided confidence interval for \({ \beta }_{ j }\). The implication here is that the true value of \({ \beta }_{ j }\) is contained in 95% of all possible randomly drawn variables.

Alternatively, the 95% two-sided confidence interval for \({ \beta }_{ j }\) is the set of values that are impossible to reject when a two-sided hypothesis test of 5% is applied. Therefore, with a large sample size:

$$ 95\%\quad confidence\quad interval\quad for\quad { \beta }_{ j }=\left[ { \hat { \beta } }_{ j }-1.96SE\left( { \hat { \beta } }_{ j } \right) ,{ \hat { \beta } }_{ j }+1.96SE\left( { \hat { \beta } }_{ j } \right) \right] $$

Tests of Joint Hypotheses

In this section, we consider the formulation of the joint hypotheses on multiple regression coefficients. We will further study the application of an \(F\)-statistic in their testing.

Hypotheses Testing on Two or More Coefficients

Joint null hypothesis.

In multiple regression, we canno t test the null hypothesis that all slope coefficients are equal 0 based on t -tests that each individual slope coefficient equals 0. Why? individual t-tests do not account for the effects of interactions among the independent variables.

For this reason, we conduct the F-test which uses the F-statistic .  The F-test tests the null hypothesis that all of the slope coefficients in the multiple regression model are jointly equal to 0, .i.e.,

\(F\)-Statistic

The F-statistic, which is always a one-tailed test , is calculated as:

hypothesis test for multiple linear regression

To determine whether at least one of the coefficients is statistically significant, the calculated F-statistic is compared with the one-tailed critical F-value, at the appropriate level of significance.

Decision rule:

hypothesis test for multiple linear regression

Rejection of the null hypothesis at a stated level of significance indicates that at least one of the coefficients is significantly different than zero, i.e, at least one of the independent variables in the regression model makes a significant contribution to the dependent variable.

An analyst runs a regression of monthly value-stock returns on four independent variables over 48 months.

The total sum of squares for the regression is 360, and the sum of squared errors is 120.

Test the null hypothesis at the 5% significance level (95% confidence) that all the four independent variables are equal to zero.

\({ H }_{ 0 }:{ \beta }_{ 1 }=0,{ \beta }_{ 2 }=0,\dots ,{ \beta }_{ 4 }=0 \)

\({ H }_{ 1 }:{ \beta }_{ j }\neq 0\) (at least one j is not equal to zero, j=1,2… k )

ESS = TSS – SSR = 360 – 120 = 240

The calculated test statistic = (ESS/k)/(SSR/(n-k-1))

=(240/4)/(120/43) = 21.5

\({ F }_{ 43 }^{ 4 }\) is approximately 2.44 at 5% significance level.

Decision: Reject H 0 .

Conclusion: at least one of the 4 independents is significantly different than zero.

Omitted Variable Bias in Multiple Regression

This is the bias in the OLS estimator arising when at least one included regressor gets collaborated with an omitted variable. The following conditions must be satisfied for an omitted variable bias to occur:

  • There must be a correlation between at least one of the included regressors and the omitted variable.
  • The dependent variable \(Y\) must be determined by the omitted variable.

Practical Interpretation of the \({ R }^{ 2 }\) and the adjusted \({ R }^{ 2 }\), \({ \bar { R } }^{ 2 }\)

To determine the accuracy within which the OLS regression line fits the data, we apply the coefficient of determination and the regression’s standard error . 

The coefficient of determination, represented by \({ R }^{ 2 }\), is a measure of the “goodness of fit” of the regression. It is interpreted as the percentage of variation in the dependent variable explained by the independent variables

hypothesis test for multiple linear regression

\({ R }^{ 2 }\) is not a reliable indicator of the explanatory power of a multiple regression model.Why? \({ R }^{ 2 }\) almost always increases as new independent variables are added to the model, even if the marginal contribution of the new variable is not statistically significant. Thus, a high \({ R }^{ 2 }\) may reflect the impact of a large set of independents rather than how well the set explains the dependent.This problem is solved by the use of the adjusted \({ R }^{ 2 }\) (extensively covered in chapter 8)

The following are the factors to watch out when guarding against applying the \({ R }^{ 2 }\) or the \({ \bar { R } }^{ 2 }\):

  • An added variable doesn’t have to be statistically significant just because the \({ R }^{ 2 }\) or the \({ \bar { R } }^{ 2 }\) has increased.
  • It is not always true that the regressors are a true cause of the dependent variable, just because there is a high \({ R }^{ 2 }\) or \({ \bar { R } }^{ 2 }\).
  • It is not necessary that there is no omitted variable bias just because we have a high \({ R }^{ 2 }\) or \({ \bar { R } }^{ 2 }\).
  • It is not necessarily true that we have the most appropriate set of regressors just because we have a high \({ R }^{ 2 }\) or \({ \bar { R } }^{ 2 }\).
  • It is not necessarily true that we have an inappropriate set of regressors just because we have a low \({ R }^{ 2 }\) or \({ \bar { R } }^{ 2 }\).

An economist tests the hypothesis that GDP growth in a certain country can be explained by interest rates and inflation.

Using some 30 observations, the analyst formulates the following regression equation:

$$ GDP growth = { \hat { \beta } }_{ 0 } + { \hat { \beta } }_{ 1 } Interest+ { \hat { \beta } }_{ 2 } Inflation $$

Regression estimates are as follows:

Is the coefficient for interest rates significant at 5%?

  • Since the test statistic < t-critical, we accept H 0 ; the interest rate coefficient is  not   significant at the 5% level.
  • Since the test statistic > t-critical, we reject H 0 ; the interest rate coefficient is not significant at the 5% level.
  • Since the test statistic > t-critical, we reject H 0 ; the interest rate coefficient is significant at the 5% level.
  • Since the test statistic < t-critical, we accept H 1 ; the interest rate coefficient is significant at the 5% level.

The correct answer is  C .

We have GDP growth = 0.10 + 0.20(Int) + 0.15(Inf)

Hypothesis:

$$ { H }_{ 0 }:{ \hat { \beta } }_{ 1 } = 0 \quad vs \quad { H }_{ 1 }:{ \hat { \beta } }_{ 1 }≠0 $$

The test statistic is:

$$ t = \left( \frac { 0.20 – 0 }{ 0.05 } \right)  = 4 $$

The critical value is t (α/2, n-k-1) = t 0.025,27  = 2.052 (which can be found on the t-table).

t-table-25-29

Conclusion : The interest rate coefficient is significant at the 5% level.

Offered by AnalystPrep

hypothesis test for multiple linear regression

Modeling Cycles: MA, AR, and ARMA Models

Empirical approaches to risk metrics and hedging, properties of interest rates.

After completing this reading, you should be able to: Describe the various categories... Read More

Exchanges and OTC Markets

After completing this reading, you should be able to: Describe how exchanges can... Read More

Interest Rate Futures

After completing this reading, you should be able to: Identify the most commonly... Read More

Sample Moments

After completing this reading, you should be able to: Estimate the mean, variance,... Read More

Leave a Comment Cancel reply

You must be logged in to post a comment.

logo

Multiple linear regression

Multiple linear regression #.

Fig. 11 Multiple linear regression #

Errors: \(\varepsilon_i \sim N(0,\sigma^2)\quad \text{i.i.d.}\)

Fit: the estimates \(\hat\beta_0\) and \(\hat\beta_1\) are chosen to minimize the residual sum of squares (RSS):

Matrix notation: with \(\beta=(\beta_0,\dots,\beta_p)\) and \({X}\) our usual data matrix with an extra column of ones on the left to account for the intercept, we can write

Multiple linear regression answers several questions #

Is at least one of the variables \(X_i\) useful for predicting the outcome \(Y\) ?

Which subset of the predictors is most important?

How good is a linear model for these data?

Given a set of predictor values, what is a likely value for \(Y\) , and how accurate is this prediction?

The estimates \(\hat\beta\) #

Our goal again is to minimize the RSS: $ \( \begin{aligned} \text{RSS}(\beta) &= \sum_{i=1}^n (y_i -\hat y_i(\beta))^2 \\ & = \sum_{i=1}^n (y_i - \beta_0- \beta_1 x_{i,1}-\dots-\beta_p x_{i,p})^2 \\ &= \|Y-X\beta\|^2_2 \end{aligned} \) $

One can show that this is minimized by the vector \(\hat\beta\) : $ \(\hat\beta = ({X}^T{X})^{-1}{X}^T{y}.\) $

We usually write \(RSS=RSS(\hat{\beta})\) for the minimized RSS.

Which variables are important? #

Consider the hypothesis: \(H_0:\) the last \(q\) predictors have no relation with \(Y\) .

Based on our model: \(H_0:\beta_{p-q+1}=\beta_{p-q+2}=\dots=\beta_p=0.\)

Let \(\text{RSS}_0\) be the minimized residual sum of squares for the model which excludes these variables.

The \(F\) -statistic is defined by: $ \(F = \frac{(\text{RSS}_0-\text{RSS})/q}{\text{RSS}/(n-p-1)}.\) $

Under the null hypothesis (of our model), this has an \(F\) -distribution.

Example: If \(q=p\) , we test whether any of the variables is important. $ \(\text{RSS}_0 = \sum_{i=1}^n(y_i-\overline y)^2 \) $

The \(t\) -statistic associated to the \(i\) th predictor is the square root of the \(F\) -statistic for the null hypothesis which sets only \(\beta_i=0\) .

A low \(p\) -value indicates that the predictor is important.

Warning: If there are many predictors, even under the null hypothesis, some of the \(t\) -tests will have low p-values even when the model has no explanatory power.

How many variables are important? #

When we select a subset of the predictors, we have \(2^p\) choices.

A way to simplify the choice is to define a range of models with an increasing number of variables, then select the best.

Forward selection: Starting from a null model, include variables one at a time, minimizing the RSS at each step.

Backward selection: Starting from the full model, eliminate variables one at a time, choosing the one with the largest p-value at each step.

Mixed selection: Starting from some model, include variables one at a time, minimizing the RSS at each step. If the p-value for some variable goes beyond a threshold, eliminate that variable.

Choosing one model in the range produced is a form of tuning . This tuning can invalidate some of our methods like hypothesis tests and confidence intervals…

How good are the predictions? #

The function predict in R outputs predictions and confidence intervals from a linear model:

Prediction intervals reflect uncertainty on \(\hat\beta\) and the irreducible error \(\varepsilon\) as well.

These functions rely on our linear regression model $ \( Y = X\beta + \epsilon. \) $

Dealing with categorical or qualitative predictors #

For each qualitative predictor, e.g. Region :

Choose a baseline category, e.g. East

For every other category, define a new predictor:

\(X_\text{South}\) is 1 if the person is from the South region and 0 otherwise

\(X_\text{West}\) is 1 if the person is from the West region and 0 otherwise.

The model will be: $ \(Y = \beta_0 + \beta_1 X_1 +\dots +\beta_7 X_7 + \color{Red}{\beta_\text{South}} X_\text{South} + \beta_\text{West} X_\text{West} +\varepsilon.\) $

The parameter \(\color{Red}{\beta_\text{South}}\) is the relative effect on Balance (our \(Y\) ) for being from the South compared to the baseline category (East).

The model fit and predictions are independent of the choice of the baseline category.

However, hypothesis tests derived from these variables are affected by the choice.

Solution: To check whether region is important, use an \(F\) -test for the hypothesis \(\beta_\text{South}=\beta_\text{West}=0\) by dropping Region from the model. This does not depend on the coding.

Note that there are other ways to encode qualitative predictors produce the same fit \(\hat f\) , but the coefficients have different interpretations.

So far, we have:

Defined Multiple Linear Regression

Discussed how to test the importance of variables.

Described one approach to choose a subset of variables.

Explained how to code qualitative variables.

Now, how do we evaluate model fit? Is the linear model any good? What can go wrong?

How good is the fit? #

To assess the fit, we focus on the residuals $ \( e = Y - \hat{Y} \) $

The RSS always decreases as we add more variables.

The residual standard error (RSE) corrects this: $ \(\text{RSE} = \sqrt{\frac{1}{n-p-1}\text{RSS}}.\) $

Fig. 12 Residuals #

Visualizing the residuals can reveal phenomena that are not accounted for by the model; eg. synergies or interactions:

Potential issues in linear regression #

Interactions between predictors

Non-linear relationships

Correlation of error terms

Non-constant variance of error (heteroskedasticity)

High leverage points

Collinearity

Interactions between predictors #

Linear regression has an additive assumption: $ \(\mathtt{sales} = \beta_0 + \beta_1\times\mathtt{tv}+ \beta_2\times\mathtt{radio}+\varepsilon\) $

i.e. An increase of 100 USD dollars in TV ads causes a fixed increase of \(100 \beta_2\) USD in sales on average, regardless of how much you spend on radio ads.

We saw that in Fig 3.5 above. If we visualize the fit and the observed points, we see they are not evenly scattered around the plane. This could be caused by an interaction.

One way to deal with this is to include multiplicative variables in the model:

The interaction variable tv \(\cdot\) radio is high when both tv and radio are high.

R makes it easy to include interaction variables in the model:

Non-linearities #

Fig. 13 A nonlinear fit might be better here. #

Example: Auto dataset.

A scatterplot between a predictor and the response may reveal a non-linear relationship.

Solution: include polynomial terms in the model.

Could use other functions besides polynomials…

Fig. 14 Residuals for Auto data #

In 2 or 3 dimensions, this is easy to visualize. What do we do when we have too many predictors?

Correlation of error terms #

We assumed that the errors for each sample are independent:

What if this breaks down?

The main effect is that this invalidates any assertions about Standard Errors, confidence intervals, and hypothesis tests…

Example : Suppose that by accident, we duplicate the data (we use each sample twice). Then, the standard errors would be artificially smaller by a factor of \(\sqrt{2}\) .

When could this happen in real life:

Time series: Each sample corresponds to a different point in time. The errors for samples that are close in time are correlated.

Spatial data: Each sample corresponds to a different location in space.

Grouped data: Imagine a study on predicting height from weight at birth. If some of the subjects in the study are in the same family, their shared environment could make them deviate from \(f(x)\) in similar ways.

Correlated errors #

Simulations of time series with increasing correlations between \(\varepsilon_i\)

Non-constant variance of error (heteroskedasticity) #

The variance of the error depends on some characteristics of the input features.

To diagnose this, we can plot residuals vs. fitted values:

If the trend in variance is relatively simple, we can transform the response using a logarithm, for example.

Outliers from a model are points with very high errors.

While they may not affect the fit, they might affect our assessment of model quality.

Possible solutions: #

If we believe an outlier is due to an error in data collection, we can remove it.

An outlier might be evidence of a missing predictor, or the need to specify a more complex model.

High leverage points #

Some samples with extreme inputs have an outsized effect on \(\hat \beta\) .

This can be measured with the leverage statistic or self influence :

Studentized residuals #

The residual \(e_i = y_i - \hat y_i\) is an estimate for the noise \(\epsilon_i\) .

The standard error of \(\hat \epsilon_i\) is \(\sigma \sqrt{1-h_{ii}}\) .

A studentized residual is \(\hat \epsilon_i\) divided by its standard error (with appropriate estimate of \(\sigma\) )

When model is correct, it follows a Student-t distribution with \(n-p-2\) degrees of freedom.

Collinearity #

Two predictors are collinear if one explains the other well:

Problem: The coefficients become unidentifiable .

Consider the extreme case of using two identical predictors limit : $ \( \begin{aligned} \mathtt{balance} &= \beta_0 + \beta_1\times\mathtt{limit} + \beta_2\times\mathtt{limit} + \epsilon \\ & = \beta_0 + (\beta_1+100)\times\mathtt{limit} + (\beta_2-100)\times\mathtt{limit} + \epsilon \end{aligned} \) $

For every \((\beta_0,\beta_1,\beta_2)\) the fit at \((\beta_0,\beta_1,\beta_2)\) is just as good as at \((\beta_0,\beta_1+100,\beta_2-100)\) .

If 2 variables are collinear, we can easily diagnose this using their correlation.

A group of \(q\) variables is multilinear if these variables “contain less information” than \(q\) independent variables.

Pairwise correlations may not reveal multilinear variables.

The Variance Inflation Factor (VIF) measures how predictable it is given the other variables, a proxy for how necessary a variable is:

Above, \(R^2_{X_j|X_{-j}}\) is the \(R^2\) statistic for Multiple Linear regression of the predictor \(X_j\) onto the remaining predictors.

Advanced Statistics using R

Applied Data Science Meeting, July 4-6, 2023, Shanghai, China . Register for the workshops: (1) Deep Learning Using R, (2) Introduction to Social Network Analysis, (3) From Latent Class Model to Latent Transition Model Using Mplus, (4) Longitudinal Data Analysis, and (5) Practical Mediation Analysis. Click here for more information .

  • Example Datasets
  • Basics of R
  • Graphs in R

Hypothesis testing

  • Confidence interval
  • Simple Regression
  • Multiple Regression
  • Logistic regression
  • Moderation analysis
  • Mediation analysis
  • Path analysis
  • Factor analysis
  • Multilevel regression
  • Longitudinal data analysis
  • Power analysis

Multiple Linear Regression

The general purpose of multiple regression (the term was first used by Pearson, 1908), as a generalization of simple linear regression, is to learn about how several independent variables or predictors (IVs) together predict a dependent variable (DV). Multiple regression analysis often focuses on understanding (1) how much variance in a DV a set of IVs explain and (2) the relative predictive importance of IVs in predicting a DV.

In the social and natural sciences, multiple regression analysis is very widely used in research. Multiple regression allows a researcher to ask (and hopefully answer) the general question "what is the best predictor of ...". For example, educational researchers might want to learn what the best predictors of success in college are. Psychologists may want to determine which personality dimensions best predicts social adjustment.

Multiple regression model

A general multiple linear regression model at the population level can be written as

\[y_{i}=\beta_{0}+\beta_{1}x_{1i}+\beta_{2}x_{2i}+\ldots+\beta_{k}x_{ki}+\varepsilon_{i} \]

  • $y_{i}$: the observed score of individual $i$ on the DV.
  • $x_{1},x_{2},\ldots,x_{k}$ : a set of predictors.
  • $x_{1i}$: the observed score of individual $i$ on IV 1; $x_{ki}$: observed score of individual $i$ on IV $k$.
  • $\beta_{0}$: the intercept at the population level, representing the predicted $y$ score when all the independent variables have their values at 0.
  • $\beta_{1},\ldots,\beta_{k}$: regression coefficients at the population level; $\beta_{1}$: representing the amount predicted $y$ changes when $x_{1}$ changes in 1 unit while holding the other IVs constant; $\beta_{k}$: representing the amount predicted $y$ changes when $x_{k}$ changes in 1 unit while holding the other IVs constant.
  • $\varepsilon$: unobserved errors with mean 0 and variance $\sigma^{2}$.

Parameter estimation

The least squares method used for the simple linear regression analysis can also be used to estimate the parameters in a multiple regression model. The basic idea is to minimize the sum of squared residuals or errors. Let $b_{0},b_{1},\ldots,b_{k}$ represent the estimated regression coefficients.The individual $i$'s residual $e_{i}$ is the difference between the observed $y_{i}$ and the predicted $y_{i}$

\[ e_{i}=y_{i}-\hat{y}_{i}=y_{i}-b_{0}-b_{1}x_{1i}-\ldots-b_{k}x_{ki}.\]

The sum of squared residuals is

\[ SSE=\sum_{i=1}^{n}e_{i}^{2}=\sum_{i=1}^{n}(y_{i}-\hat{y}_{i})^{2}. \]

By minimizing $SSE$, the regression coefficient estimates can be obtained as

\[ \boldsymbol{b}=(\boldsymbol{X}'\boldsymbol{X})^{-1}\boldsymbol{X}'\boldsymbol{y}=(\sum\boldsymbol{x}_{i}\boldsymbol{x}_{i}')^{-1}(\sum\boldsymbol{x}_{i}\boldsymbol{y}_{i}). \]

How well the multiple regression model fits the data can be assessed using the $R^{2}$. Its calculation is the same as for the simple regression

\[\begin{align*} R^{2} & = & 1-\frac{\sum e_{i}^{2}}{\sum_{i=1}^{n}(y_{i}-\bar{y})^{2}}\\& = & \frac{\text{Variation explained by IVs}}{\text{Total variation}} \end{align*}. \]

In multiple regression, $R^{2}$ is the total proportion of variation in $y$ explained by the multiple predictors.

The $R^{2}$ increases or at least is the same with the inclusion of more predictors. However, with more predators, the model becomes more complex and potentially more difficult to interpret. In order to take into consideration of the model complexity, the adjusted $R^{2}$ has been defined, which is calculated as

\[aR^{2}=1-(1-R^{2})\frac{n-1}{n-k-1}.\]

Hypothesis testing of regression coefficient(s)

With the estimates of regression coefficients and their standard errors estimates, we can conduct hypothesis testing for one, a subset, or all regression coefficients.

Testing a single regression coefficient

At first, we can test the significance of the coefficient for a single predictor. In this situation, the null and alternative hypotheses are

\[ H_{0}:\beta_{j}=0\text{ vs }H_{1}:\beta_{j}\neq0 \]

with $\beta_{j}$ denoting the regression coefficient of $x_{j}$ at the population level.

As in the simple regression, we use a test statistic

\[ t_{j}=\frac{b_{j} - \beta{j} }{s.e.(b_{j})}\]

where $b_{j}$ is the estimated regression coefficient of $x_{j}$ using data from a sample. If the null hypothesis is true and $\beta_j = 0$, the test statistic follows a t-distribution with degrees of freedom \(n-k-1\) where \(k\) is the number of predictors.

One can also test the significance of \(\beta_j\) by constructing a confidence interval for it. Based on a t distribution, the \(100(1-\alpha)%\) confidence interval is

\[ [b_{j}+t_{n-k-1}(\alpha/2)*s.e.(b_{j}),\;b_{j}+t_{n-k-1}(1-\alpha/2)*s.e.(b_{j})]\]

where $t_{n-k-1}(\alpha/2)$ is the $\alpha/2$ percentile of the t distribution. As previously discussed, if the confidence interval includes 0, the regression coefficient is not statistically significant at the significance level $\alpha$.

Testing all the regression coefficients together (overall model fit)

Given the multiple predictors, we can also test whether all of the regression coefficients are 0 at the same time. This is equivalent to test whether all predictors combined can explained a significant portion of the variance of the outcome variable. Since $R^2$ is a measure of the variance explained, this test is naturally related to it.

For this hypothesis testing, the null and alternative hypothesis are

\[H_{0}:\beta_{1}=\beta_{2}=\ldots=\beta_{k}=0\]

\[H_{1}:\text{ at least one of the regression coefficients is different from 0}.\]

In this kind of test, an F test is used. The F-statistic is defined as

\[F=\frac{n-k-1}{k}\frac{R^{2}}{1-R^{2}}.\]

It follows an F-distribution with degrees of freedom $k$ and $n-k-1$ when the null hypothesis is true. Given an F statistic, its corresponding p-value can be calculated from the F distribution as shown below. Note that we only look at one side of the distribution because the extreme values should be on the large value side.

Testing a subset of the regression coefficients

We can also test whether a subset of $p$ regression coefficients, e.g., $p$ from 1 to the total number coefficients $k$, are equal to zero. For convenience, we can rearrange all the $p$ regression coefficients to be the first $p$ coefficients. Therefore, the null hypothesis should be

\[H_{0}:\beta_{1}=\beta_{2}=\ldots=\beta_{p}=0\]

and the alternative hypothesis is that at least one of them is not equal to 0.

As for testing the overall model fit, an F test can be used here. In this situation, the F statistic can be calculated as

\[F=\frac{n-k-1}{p}\frac{R^{2}-R_{0}^{2}}{1-R^{2}},\]

which follows an F-distribution with degrees of freedom $p$ and $n-k-1$. $R^2$ is for the regression model with all the predictors and $R_0^2$ is from the regression model without the first $p$ predictors $x_{1},x_{2},\ldots,x_{p}$ but with the rest predictors $x_{p+1},x_{p+2},\ldots,x_{k}$.

Intuitively, this test determine whether the variance explained by the first \(p\) predictors above and beyond the $k-p$ predictors is significance or not. That is also the increase in R-squared.

As an example, suppose that we wanted to predict student success in college. Why might we want to do this? There's an ongoing debate in college and university admission offices (and in the courts) regarding what factors should be considered important in deciding which applicants to admit. Should admissions officers pay most attention to more easily quantifiable measures such as high school GPA and SAT scores? Or should they give more weight to more subjective measures such as the quality of letters of recommendation? What are the pros and cons of the approaches? Of course, how we define college success is also an open question. For the sake of this example, let's measure college success using college GPA.

In this example, we use a set of simulated data (generated by us). The data are saved in the file gpa.csv. As shown below, the sample size is 100 and there are 4 variables: college GPA (c.gpa), high school GPA (h.gpa), SAT, and quality of recommendation letters (recommd).

Graph the data

Before fitting a regression model, we should check the relationship between college GPA and each predictor through a scatterplot. A scatterplot can tell us the form of relationship, e.g., linear, nonlinear, or no relationship, the direction of relationship, e.g., positive or negative, and the strength of relationship, e.g., strong, moderate, or weak. It can also identify potential outliers.

The scatterplots between college GPA and the three potential predictors are given below. From the plots, we can roughly see all three predictors are positively related to the college GPA. The relationship is close to linear and the relationship seems to be stronger for high school GPA and SAT than for the quality of recommendation letters.

Descriptive statistics

Next, we can calculate some summary statistics to explore our data further. For each variable, we calculate 6 numbers: minimum, 1st quartile, median, mean, 3rd quartile, and maximum. Those numbers can be obtained using the summary() function. To look at the relationship among the variables, we can calculate the correlation matrix using the correlation function cor() .

Based on the correlation matrix, the correlation between college GPA and high school GPA is about 0.545, which is larger than that (0.523) between college GPA and SAT, in turn larger than that (0.35) between college GPA and quality of recommendation letters.

Fit a multiple regression model

As for the simple linear regression, The multiple regression analysis can be carried out using the lm() function in R. From the output, we can write out the regression model as

\[ c.gpa = -0.153+ 0.376 \times h.gpa + 0.00122 \times SAT + 0.023 \times recommd \]

Interpret the results / output

From the output, we see the intercept is -0.153. Its immediate meaning is that when all predictors' values are 0, the predicted college GPA is -0.15. This clearly does not make much sense because one would never get a negative GPA, which results from the unrealistic presumption that the predictors can take the value of 0.

The regression coefficient for the predictor high school GPA (h.gpa) is 0.376. This can be interpreted as keeping SAT and recommd scores constant , the predicted college GPA would increase 0.376 with a unit increase in high school GPA.This is again might be problematic because it might be impossible to increase high school GPA while keeping the other two predictors unchanged. The other two regression coefficients can be interpreted in the same way.

From the output, we can also see that the multiple R-squared ($R^2$) is 0.3997. Therefore, about 40% of the variation in college GPA can be explained by the multiple linear regression with h.GPA, SAT, and recommd as the predictors. The adjusted $R^2$ is slightly smaller because of the consideration of the number of predictors. In fact,

\[ \begin{eqnarray*} aR^{2} & = & 1-(1-R^{2})\frac{n-1}{n-k-1}\\& = & 1-(1-.3997)\frac{100-1}{100-3-1}\\& = & .3809 \end{eqnarray*} \]

Testing Individual Regression Coefficient

For any regression coefficients for the three predictors (also the intercept), a t test can be conducted. For example, for high school GPA, the estimated coefficient is 0.376 with the standard error 0.114. Therefore, the corresponding t statistic is \(t = 0.376/0.114 = 3.294\). Since the statistic follows a t distribution with the degrees of freedom \(df = n - k - 1 = 100 - 3 -1 =96\), we can obtain the p-value as \(p = 2*(1-pt(3.294, 96))= 0.0013\). Since the p-value is less than 0.05, we conclude the coefficient is statistically significant. Note the t value and p-value are directly provided in the output.

Overall model fit (testing all coefficients together)

To test all coefficients together or the overall model fit, we use the F test. Given the $R^2$, the F statistic is

\[ \begin{eqnarray*} F & = & \frac{n-k-1}{k}\frac{R^{2}}{1-R^{2}}\\ & = & \left(\frac{100-3-1}{3}\right)\times \left(\frac{0.3997}{1-.3997}\right )=21.307\end{eqnarray*} \]

which follows the F distribution with degrees of freedom $df1=k=3$ and $df2=n-k-1=96$. The corresponding p-value is 1.160e-10. Note that this information is directly shown in the output as " F-statistic: 21.31 on 3 and 96 DF, p-value: 1.160e-10 ".

Therefore, at least one of the regression coefficients is statistically significantly different from 0. Overall, the three predictors explained a significant portion of the variance in college GPA. The regression model with the 3 predictors is significantly better than the regression model with intercept only (i.e., predict c.gpa by the mean of c.gpa).

Testing a subset of regression coefficients

Suppose we are interested in testing whether the regression coefficients of high school GPA and SAT together are significant or not. Alternative, we want to see above and beyond the quality of recommendation letters, whether the two predictors can explain a significant portion of variance in college GPA. To conduct the test, we need to fit two models:

  • A full model: which consists of all the predictors to predict c.gpa by intercept, h.gpa, SAT, and recommd.
  • A reduced model: obtained by removing the predictors to be tested in the full model.

From the full model, we can get the $R^2 = 0.3997$ with all three predictors and from the reduced model, we can get the $R_0^2 = 0.1226$ with only quality of recommendation letters. Then the F statistic is constructed as

\[F=\frac{n-k-1}{p}\frac{R^{2}-R_{0}^{2}}{1-R^{2}}=\left(\frac{100-3-1}{2}\right )\times\frac{.3997-.1226}{1-.3997}=22.157.\]

Using the F distribution with the degrees of freedom $p=2$ (the number of coefficients to be tested) and $n-k-1 = 96$, we can get the p-value close to 0 ($p=1.22e-08$).

Note that the test conducted here is based on the comparison of two models. In R, if there are two models, they can be compared conveniently using the R function anova() . As shown below, we obtain the same F statistic and p-value.

To cite the book, use: Zhang, Z. & Wang, L. (2017-2022). Advanced statistics using R . Granger, IN: ISDSA Press. https://doi.org/10.35566/advstats. ISBN: 978-1-946728-01-2. To take the full advantage of the book such as running analysis within your web browser, please subscribe .

Library homepage

  • school Campus Bookshelves
  • menu_book Bookshelves
  • perm_media Learning Objects
  • login Login
  • how_to_reg Request Instructor Account
  • hub Instructor Commons
  • Download Page (PDF)
  • Download Full Book (PDF)
  • Periodic Table
  • Physics Constants
  • Scientific Calculator
  • Reference & Cite
  • Tools expand_more
  • Readability

selected template will load here

This action is not available.

Statistics LibreTexts

8.7: Overall F-test in multiple linear regression

  • Last updated
  • Save as PDF
  • Page ID 33297

  • Mark Greenwood
  • Montana State University

In the MLR summary, there is an \(F\) -test and p-value reported at the bottom of the output. For the model with Elevation and Maximum Temperature , the last row of the model summary is:

This test is called the overall F-test in MLR and is very similar to the \(F\) -test in a reference-coded One-Way ANOVA model. It tests the null hypothesis that involves setting every coefficient except the \(y\) -intercept to 0 (so all the slope coefficients equal 0). We saw this reduced model in the One-Way material when we considered setting all the deviations from the baseline group to 0 under the null hypothesis. We can frame this as a comparison between a full and reduced model as follows:

  • Full Model: \(y_i = \beta_0 + \beta_1x_{1i} + \beta_2x_{2i}+\cdots + \beta_Kx_{Ki}+\varepsilon_i\)
  • Reduced Model: \(y_i = \beta_0 + 0x_{1i} + 0x_{2i}+\cdots + 0x_{Ki}+\varepsilon_i\)

The reduced model estimates the same values for all \(y\text{'s}\) , \(\widehat{y}_i = \bar{y} = b_0\) and corresponds to the null hypothesis of:

\(\boldsymbol{H_0:}\) No explanatory variables should be included in the model: \(\beta_1 = \beta_2 = \cdots = \beta_K = 0\) .

The full model corresponds to the alternative:

\(\boldsymbol{H_A:}\) At least one explanatory variable should be included in the model: Not all \(\beta_k\text{'s} = 0\) for \((k = 1,\ldots,K)\) .

Note that \(\beta_0\) is not set to 0 in the reduced model (under the null hypothesis) – it becomes the true mean of \(y\) for all values of the \(x\text{'s}\) since all the predictors are multiplied by coefficients of 0.

The test statistic to assess these hypotheses is \(F = \text{MS}_{\text{model}}/\text{MS}_E\) , which is assumed to follow an \(F\) -distribution with \(K\) numerator df and \(n-K-1\) denominator df under the null hypothesis. The output provides us with \(F(2, 20) = 56.43\) and a p-value of \(5.979*10^{-9}\) (p-value \(<0.00001\) ) and strong evidence against the null hypothesis. Thus, there is strong evidence against the null hypothesis that the true slopes for the two predictors are 0 and so we would conclude that at least one of the two slope coefficients ( Max.Temp ’s or Elevation ’s) is different from 0 in the population of SNOTEL sites in Montana on this date. While this test is a little bit interesting and a good indicator of something interesting existing in the model, the moment you see this result, you want to know more about each predictor variable. If neither predictor variable is important, we will discover that in the \(t\) -tests for each coefficient and so our general recommendation is to start there.

The overall F-test, then, is really about testing whether there is something good in the model somewhere. And that certainly is important but it is also not too informative. There is one situation where this test is really interesting, when there is only one predictor variable in the model (SLR). In that situation, this test provides exactly the same p-value as the \(t\) -test. \(F\) -tests will be important when we are mixing categorical and quantitative predictor variables in our MLR models (Section 8.12), but the overall \(F\) -test is of very limited utility.

Multiple Linear Regression

We hope you enjoyed this lesson.

Get the Hypothesis Testing course for more great video tutorials.

Start free trial

Cool lesson, huh? Share it with your friends

Facebook Twitter LinkedIn WhatsApp Email

  • Lesson resources Resources
  • Quick reference Reference

About this lesson

Many times there are multiple factors that are influencing the response variable in a problem. Multiple regression determines the relationship between the response factor and multiple control factors. Like with simple linear regression, a formula is created that allows both analysis and prediction of the process and problem.

Exercise files

Download this lesson’s related exercise files.

Quick reference

Multiple linear regression analysis is the creation of an equation with multiple independent X variables that all influence a Y response variable.  This equation is based upon an existing data set and models the conditions represented in the data.

When to use

When there are multiple independent variables that correlate with the system response, a multiple linear regression should be done.  This can be used to predict process performance and identify which factors have the primary impact on process performance.

Instructions

Multiple linear regression is the appropriate technique to use when the data set has multiple continuous independent input variables and a continuous response variable.  The technique determines which variables are statistically significant and creates an equation that shows the relationship of the variables to the response.  To improve the accuracy of the analysis, there should be at least ten data points for each independent variable.  The equation takes on the form:

Y = a + b 1 X 1 + b 2 X 2 + b 3 X 3 + …

Where the absolute value of the “b” coefficients shows the relative importance of each variable.

Multiple linear regression can be used to predict process performance based on the values of the inputs.  Input levels for ideal performance can be defined and tolerance levels that ensure acceptable performance can be determined using the regression equation.  The equation will also be helpful for setting process controls.

Excel does not have a multiple linear regression function.  The analysis can be done in Minitab using the “Fit Regression Model” option in the Regression menu.  This will display an input panel where the response variable and input variables can be selected.  If the analysis shows a variable is not statistically significant, check the residual plots to see if the result is normal.  If not, remove the variable that is not statistically significant and rerun the analysis.  The normality of the residuals should be improved.

Hints & tips

  • Too many variables increase uncertainty in the analysis.  There should be at least ten data points for each variable (e.g. if using three variables have at least 30 data points). 
  • Drop variables that are not statistically significant to improve the accuracy of the equation.
  • The analysis assumes a linear (straight line) effect.  If the residuals indicate a bad fit, you will need to add higher-order terms and create a non-linear analysis.  This is discussed in another lesson.
  • Always check the residual analysis to ensure it is normally distributed with equal variance and indicates independence.
  • 00:04 Hi, I'm Ray Sheen.
  • 00:05 Sometimes the dependent or
  • 00:07 response variable in the analysis depends upon more than one factor.
  • 00:12 When that happens, you may need to do a multi-linear regression analysis.
  • 00:18 >> Once again, I'll start with our decision tree for hypothesis testing.
  • 00:22 When we have continuous variables for the process response and
  • 00:26 the process independent variables, we turn to the regression analysis, and when we
  • 00:30 have multiple variables at the same time, we use the multiple regression technique.
  • 00:36 Let's take a few minutes to explain what we mean by multiple regression analysis.
  • 00:42 Recall that regression analysis determines the relationship between process
  • 00:45 variables.
  • 00:47 And it's no surprise that multiple regression considers multiple independent
  • 00:51 variables instead of just one.
  • 00:53 The analysis determines the impact of each of these independent
  • 00:57 variables upon the dependent variable.
  • 01:00 If you tried a simple linear analysis and it wasn't a good fit,
  • 01:03 you can consider adding some additional terms.
  • 01:07 Now, regardless of the number of terms, the format for
  • 01:09 the hypothesis tests are still the same as a simple linear regression.
  • 01:13 The null hypothesis is that there is no relationship, and
  • 01:17 the alternative hypothesis is that there is a relationship.
  • 01:21 The analysis will determine the relative significance of each of the factors to
  • 01:25 each other in addition to the dependent factor.
  • 01:28 It will show up with coefficients of these factors.
  • 01:31 The form of the multiple linear equation is a dependent variable y is equal to
  • 01:36 a constant indicated by a in this equation plus a term with each of the variables,
  • 01:41 which has a coefficient associated with it.
  • 01:44 So it's beta 1 times x1 + beta 2 times x 2 + beta 3 times x 3 and so on.
  • 01:53 Multiple regression analysis is particularly useful for
  • 01:56 predicting process performance.
  • 01:59 The multiple regression analysis will result in an equation that relates all
  • 02:03 the independent variables to the dependent variable.
  • 02:07 This analysis provides the terms and the coefficients.
  • 02:12 This equation is incredibly helpful when you're designing the solution for
  • 02:15 a problem in a Lean Six Sigma Project.
  • 02:18 The equation predicts the dependent variable performance based upon whatever
  • 02:22 values you've selected for the independent variables.
  • 02:25 So, as you're designing the solution, you may want to design one of your independent
  • 02:30 variables to be in a particular zone that's well controlled.
  • 02:34 This will then help you determine how with the other variables you can achieve
  • 02:38 the desired performance.
  • 02:40 Based upon the scaling constants for each of the factors,
  • 02:44 you can also decide which factors will have the primary control for the process.
  • 02:49 I prefer to use one easily controlled independent factor to control
  • 02:53 an overall process, and if possible to set the other factors and
  • 02:58 zones that are very easy to lock into a standard setting.
  • 03:02 You can't always do that, but if you can, it makes process control much easier.
  • 03:07 So let's look at how we conduct a multiple regression analysis.
  • 03:11 Excel does not have a function for
  • 03:13 conducting multiple linear regression analysis.
  • 03:16 So we will have to rely on Minitab.
  • 03:19 In Minitab, go to the stat pulldown menu, select Regression, select
  • 03:24 Regression again, and then select Fit Regression Model just like is shown here.
  • 03:30 That will bring up this panel.
  • 03:32 Place your cursor in the response window to activate the list of
  • 03:36 data values in the window on the left ,then select the dependent variable
  • 03:41 often referred to as the y factor and click on the selection button.
  • 03:46 The column should move to the response window.
  • 03:48 Now, place your cursor in the continuous predictors and
  • 03:52 then select the appropriate column.
  • 03:54 You can also use categorical or discrete factors if you have them.
  • 03:58 However, if you're using these factors I always recommend you use factors that
  • 04:02 are bimodal such as true, false.
  • 04:04 Set one of those to a value of 1 and the other to a value of 0.
  • 04:08 And one more point, you get the residual plots by selecting the graph button and
  • 04:13 then choosing residual 4 and 1.
  • 04:17 Let's finish off this topic with a few warnings and
  • 04:20 some pitfalls when doing multiple regression analysis.
  • 04:24 This analysis still assumes linear effect, which means straight line effects for
  • 04:28 each of the independent variables.
  • 04:31 We'll look at interactive effects when we look at nonlinear regression in
  • 04:34 another lesson.
  • 04:36 Adding lots of independent variables can increase uncertainty.
  • 04:40 If you find that some factor has virtually no effect,
  • 04:43 then I recommend removing it from your analysis and simplify things.
  • 04:48 Check your residual plots to make sure the residuals are normally distributed.
  • 04:53 This is another indication that you have a good solution.
  • 04:56 And finally, too many factors creates too many potential interactions,
  • 05:00 and it becomes difficult to statistically validate the effect of each independent
  • 05:04 variable.
  • 05:05 A good rule of thumb is that your data set size should have at least 10 times,
  • 05:10 the number of independent factors being analyzed.
  • 05:14 So if you want to analyze four factors at once,
  • 05:17 the data set needs to have a minimum of 40 points.
  • 05:20 Also, when there are many independent factors in the analysis,
  • 05:24 the regression formula becomes much more sensitive to outliers.
  • 05:29 >> In many cases,
  • 05:30 the multiple linear regression analysis is just what is needed to understand
  • 05:34 how the handful of independent variables affects the overall process output.
  • 05:40 The formula created is extremely helpful when
  • 05:44 determining the optimal solution for your problem.

Lesson notes are only available for subscribers.

PMI, PMP, CAPM and PMBOK are registered marks of the Project Management Institute, Inc.

© 2024 GoSkills Ltd. Skills for career advancement

  • How It Works

Multiple Linear Regression in SPSS

Discover Multiple Linear Regression in SPSS ! Learn how to perform, understand SPSS output , and report results in APA style. Check out this simple, easy-to-follow guide below for a quick read!

Struggling with the Multiple Regression in SPSS ? We’re here to help . We offer comprehensive assistance to students , covering assignments , dissertations , research, and more. Request Quote Now !

hypothesis test for multiple linear regression

Introduction

Welcome to our comprehensive guide on Multiple Linear Regression in SPSS . In the dynamic world of statistics, understanding the nuances of Multiple Linear Regression is key for researchers and analysts seeking a deeper understanding of relationships within their data. This blog post is your roadmap to mastering Multiple Linear Regression using the Statistical Package for the Social Sciences (SPSS).

From unraveling the fundamentals to providing practical insights through examples, this guide aims to demystify the complexities, making Multiple Linear Regression accessible to both beginners and seasoned data enthusiasts.

Definition: Multiple Linear Regression

Multiple Linear Regression expands upon the principles of Simple Linear Regression by accommodating multiple independent variables. In essence, it assesses the linear relationship between the dependent variable and two or more predictors. The model’s flexibility allows for a more realistic representation of real-world scenarios where outcomes are influenced by multiple factors. By incorporating multiple predictors, this technique offers a nuanced understanding of how each variable contributes to the variation in the dependent variable. This section serves as a gateway to the intricacies of Multiple Linear Regression, setting the stage for a detailed exploration of its components and applications in subsequent sections.

Linear Regression Methods

Multiple Linear Regression encompasses various methods for building and refining models to predict a dependent variable based on multiple independent variables. These methods help researchers and analysts tailor regression models to the specific characteristics of their data and research questions. Here are some key methods in Multiple Linear Regression:

Ordinary Least Squares (OLS)

OLS is the most common method used in Multiple Linear Regression . It minimizes the sum of squared differences between observed and predicted values, aiming to find the coefficients that best fit the data. OLS provides unbiased estimates if the assumptions of the regression model are met.

Stepwise Regression

In stepwise regression , the model-building process involves adding or removing predictor variables at each step based on statistical criteria. The algorithm evaluates variables and decides whether to include or exclude them in a stepwise manner. It can be forward (adding variables) or backward (removing variables) stepwise regression.

Backward Regression

Backward regression begins with a model that includes all predictor variables and then systematically removes the least significant variables based on statistical tests. This process continues until the model only contains statistically significant predictors. It’s a simplification approach aimed at retaining only the most influential variables.

Forward Regression

Forward regression starts with an empty model and incrementally adds the most significant predictor variables based on statistical tests. This iterative process continues until the addition of more variables does not significantly improve the model. Forward regression helps identify the most relevant predictors contributing to the model’s explanatory power.

Hierarchical Regression

In hierarchical regression, predictor variables are entered into the model in a pre-defined sequence or hierarchy. This method allows researchers to examine the impact of different sets of variables on the dependent variable, taking into account their hierarchical or logical order. The most common approach involves entering blocks of variables at different steps, and assessing how each set contributes to the overall predictive power of the model.

Understanding these multiple linear regression types is crucial for selecting the most appropriate model-building strategy based on the specific goals of your analysis and the characteristics of your dataset. Each approach has its advantages and considerations, influencing the interpretability and complexity of the final regression model.

Regression Equation

The Multiple Regression Equation in Multiple Linear Regression takes the form of

Y = b0 + b1X1 + b2X2 + … + bnXn , where

  • Y is the predicted value of the dependent variable,
  • b0 is the intercept,
  • b1, b2, …, bn are the regression coefficients for each independent variable (X1, X2, …, Xn).

The regression coefficients represent the change in the dependent variable for a one-unit change in the corresponding independent variable, while the intercept is the predicted value when all independent variables are zero. Understanding the interplay between these components is essential for deciphering the impact of each predictor on the overall model. In the upcoming sections, we’ll delve deeper into specific aspects of Multiple Linear Regression, such as the role of dummy variables and the critical assumptions that underpin this statistical method.

What are Dummy Variables?

In the realm of Multiple Linear Regression , dummy variables are pivotal when dealing with categorical predictors. These variables allow us to include categorical data, like gender or region, in our regression model. Consider a binary categorical variable, such as gender (Male/Female). We represent this in our equation using a dummy variable, where one category is assigned 0 and the other 1. For instance, if Male is our reference category, the dummy variable would be 1 for Female and 0 for Male. This inclusion of categorical information enhances the model’s flexibility, capturing the nuanced impact of different categories on the dependent variable. As we explore Multiple Linear Regression further, understanding the role of dummy variables becomes paramount for robust and accurate analyses.

Assumption of Multiple Linear Regression

Before diving into Multiple Linear Regression analysis, it’s crucial to be aware of the underlying assumptions that bolster the reliability of the results.

  • Linearity : Assumes a linear relationship between the dependent variable and all independent variables. The model assumes that changes in the dependent variable are proportional to changes in the independent variables.
  • Independence of Residuals : Assumes that the residuals (the differences between observed and predicted values) are independent of each other. The independence assumption is crucial to avoid issues of autocorrelation and ensure the reliability of the model.
  • Homoscedasticity : Assumes that the variability of the residuals remains constant across all levels of the independent variables. Homoscedasticity ensures that the spread of residuals is consistent, indicating that the model’s predictions are equally accurate across the range of predictor values.
  • Normality of Residuals : Assumes that the residuals follow a normal distribution. Normality is essential for making valid statistical inferences and hypothesis testing. Deviations from normality may impact the accuracy of confidence intervals and p-values.
  • No Perfect Multicollinearity : Assumes that there is no perfect linear relationship among the independent variables. Perfect multicollinearity can lead to unstable estimates of regression coefficients, making it challenging to discern the individual impact of each predictor.

These assumptions collectively form the foundation of Multiple Linear Regression analysis . Ensuring that these conditions are met enhances the validity and reliability of the statistical inferences drawn from the model. In the subsequent sections, we will delve into hypothesis testing in Multiple Linear Regression, provide practical examples, and guide you through the step-by-step process of performing and interpreting Multiple Linear Regression analyses using SPSS.

Hypothesis of Multiple Linear Regression

The hypothesis in Multiple Linear Regression revolves around the significance of the regression coefficients. Each coefficient corresponds to a specific predictor variable, and the hypothesis tests whether each predictor has a significant impact on the dependent variable.

  • Null Hypothesis (H0): The regression coefficients for all independent variables are simultaneously equal to zero.
  • Alternative Hypothesis (H1): At least one regression coefficient for an independent variable is not equal to zero.

The hypothesis testing in Multiple Linear Regression revolves around assessing whether the collective set of independent variables has a statistically significant impact on the dependent variable. The null hypothesis suggests no overall effect, while the alternative hypothesis asserts the presence of at least one significant relationship. This testing framework guides the evaluation of the model’s overall significance, providing valuable insights into the joint contribution of the predictor variables.

Example of Simple Multiple Regression

To illustrate the concepts of Multiple Linear Regression, let’s consider an example. Imagine you are studying the factors influencing house prices, with predictors such as square footage, number of bedrooms, and distance to the city centre. By applying Multiple Linear Regression, you can model how these factors collectively influence house prices.

The regression equation would look like:

Price = b0 + b1(square footage) + b2(number of bedrooms) + b3(distance to city centre).

Through this example, you’ll gain practical insights into how Multiple Linear Regression can untangle complex relationships and offer a comprehensive understanding of the factors affecting the dependent variable.

How to Perform Multiple Linear Regression using SPSS Statistics

hypothesis test for multiple linear regression

Step by Step: Running Regression Analysis in SPSS Statistics

Now, let’s delve into the step-by-step process of conducting the Multiple Linear Regression using SPSS Statistics .  Here’s a step-by-step guide on how to perform a Multiple Linear Regression in SPSS :

  • STEP: Load Data into SPSS

Commence by launching SPSS and loading your dataset, which should encompass the variables of interest – a categorical independent variable. If your data is not already in SPSS format, you can import it by navigating to File > Open > Data and selecting your data file.

  • STEP: Access the Analyze Menu

In the top menu, locate and click on “ Analyze .” Within the “Analyze” menu, navigate to “ Regression ” and choose ” Linear ” Analyze > Regression> Linear

  • STEP: Choose Variables

A dialogue box will appear. Move the dependent variable (the one you want to predict) to the “ Dependen t” box and the independent variables to the “ Independent ” box.

  • STEP: Generate SPSS Output

Once you have specified your variables and chosen options, click the “ OK ” button to perform the analysis. SPSS will generate a comprehensive output, including the requested frequency table and chart for your dataset.

Executing these steps initiates the Multiple Linear Regression in SPSS, allowing researchers to assess the impact of the teaching method on students’ test scores while considering the repeated measures. In the next section, we will delve into the interpretation of SPSS output for Multiple Linear Regression .

Conducting a Multiple Linear Regression in SPSS provides a robust foundation for understanding the key features of your data. Always ensure that you consult the documentation corresponding to your SPSS version, as steps might slightly differ based on the software version in use. This guide is tailored for SPSS version 25 , and for any variations, it’s recommended to refer to the software’s documentation for accurate and updated instructions.

SPSS Output for Multiple Regression Analysis

hypothesis test for multiple linear regression

How to Interpret SPSS Output of Multiple Regression

Deciphering the SPSS output of Multiple Linear Regression is a crucial skill for extracting meaningful insights. Let’s focus on three tables in SPSS output;

Model Summary Table

  • R (Correlation Coefficient): This value ranges from -1 to 1 and indicates the strength and direction of the linear relationship. A positive value signifies a positive correlation, while a negative value indicates a negative correlation.
  • R-Square (Coefficient of Determination) : Represents the proportion of variance in the dependent variable explained by the independent variable. Higher values indicate a better fit of the model.
  • Adjusted R Square : Adjusts the R-squared value for the number of predictors in the model, providing a more accurate measure of goodness of fit.

ANOVA Table

  • F (ANOVA Statistic): Indicates whether the overall regression model is statistically significant. A significant F-value suggests that the model is better than a model with no predictors.
  • df (Degrees of Freedom): Represents the degrees of freedom associated with the F-test.
  • P values : The probability of obtaining the observed F-statistic by random chance. A low p-value (typically < 0.05) indicates the model’s significance.

Coefficient Table

  • Unstandardized Coefficients (B): Provides the individual regression coefficients for each predictor variable.
  • Standardized Coefficients (Beta): Standardizes the coefficients, allowing for a comparison of the relative importance of each predictor.
  • t-values : Indicate how many standard errors the coefficients are from zero. Higher absolute t-values suggest greater significance.
  • P values : Test the null hypothesis that the corresponding coefficient is equal to zero. A low p-value suggests that the predictors are significantly related to the dependent variable.

Understanding these tables in the SPSS output is crucial for drawing meaningful conclusions about the strength, significance, and direction of the relationship between variables in a Simple Linear Regression analysis.

  How to Report Results of Multiple Linear Regression in APA

Effectively communicating the results of Multiple Linear Regression in compliance with the American Psychological Association (APA) guidelines is crucial for scholarly and professional writing

  • Introduction : Begin the report with a concise introduction summarizing the purpose of the analysis and the relationship being investigated between the variables.
  • Assumption Checks: If relevant, briefly mention the checks for assumptions such as linearity, independence, homoscedasticity, and normality of residuals to ensure the robustness of the analysis.
  • Significance of the Model : Comment on the overall significance of the model based on the ANOVA table. For example, “The overall regression model was statistically significant (F = [value], p = [value]), suggesting that the predictors collectively contributed to the prediction of the dependent variable.”
  • Regression Equation : Present the Multiple Regression equation, highlighting the intercept and regression coefficients for each predictor variable.
  • Interpretation of Coefficients : Interpret the coefficients, focusing on the slope (b1..bn) to explain the strength and direction of the relationship. Discuss how a one-unit change in the independent variable corresponds to a change in the dependent variable.
  • R-squared Value: Include the R-squared value to highlight the proportion of variance in the dependent variable explained by the independent variables. For instance, “The R-squared value of [value] indicates that [percentage]% of the variability in [dependent variable] can be explained by the linear relationship with [independent variables].”
  • Conclusion : Conclude the report by summarizing the key findings and their implications. Discuss any practical significance of the results in the context of your study.

hypothesis test for multiple linear regression

Get Help For Your SPSS Analysis

Embark on a seamless research journey with SPSSAnalysis.com , where our dedicated team provides expert data analysis assistance for students, academicians, and individuals. We ensure your research is elevated with precision. Explore our pages;

  • SPSS Data Analysis Help – SPSS Helper ,
  • Quantitative Analysis Help ,
  • Qualitative Analysis Help ,
  • SPSS Dissertation Analysis Help ,
  • Dissertation Statistics Help ,
  • Statistical Analysis Help ,
  • Medical Data Analysis Help .

Connect with us at SPSSAnalysis.com to empower your research endeavors and achieve impactful results. Get a Free Quote Today !

Expert SPSS data analysis assistance available.

Struggling with Statistical Analysis in SPSS? - Hire a SPSS Helper Now!

Statology

Statistics Made Easy

Understanding the Null Hypothesis for Linear Regression

Linear regression is a technique we can use to understand the relationship between one or more predictor variables and a response variable .

If we only have one predictor variable and one response variable, we can use simple linear regression , which uses the following formula to estimate the relationship between the variables:

ŷ = β 0 + β 1 x

  • ŷ: The estimated response value.
  • β 0 : The average value of y when x is zero.
  • β 1 : The average change in y associated with a one unit increase in x.
  • x: The value of the predictor variable.

Simple linear regression uses the following null and alternative hypotheses:

  • H 0 : β 1 = 0
  • H A : β 1 ≠ 0

The null hypothesis states that the coefficient β 1 is equal to zero. In other words, there is no statistically significant relationship between the predictor variable, x, and the response variable, y.

The alternative hypothesis states that β 1 is not equal to zero. In other words, there is a statistically significant relationship between x and y.

If we have multiple predictor variables and one response variable, we can use multiple linear regression , which uses the following formula to estimate the relationship between the variables:

ŷ = β 0 + β 1 x 1 + β 2 x 2 + … + β k x k

  • β 0 : The average value of y when all predictor variables are equal to zero.
  • β i : The average change in y associated with a one unit increase in x i .
  • x i : The value of the predictor variable x i .

Multiple linear regression uses the following null and alternative hypotheses:

  • H 0 : β 1 = β 2 = … = β k = 0
  • H A : β 1 = β 2 = … = β k ≠ 0

The null hypothesis states that all coefficients in the model are equal to zero. In other words, none of the predictor variables have a statistically significant relationship with the response variable, y.

The alternative hypothesis states that not every coefficient is simultaneously equal to zero.

The following examples show how to decide to reject or fail to reject the null hypothesis in both simple linear regression and multiple linear regression models.

Example 1: Simple Linear Regression

Suppose a professor would like to use the number of hours studied to predict the exam score that students will receive in his class. He collects data for 20 students and fits a simple linear regression model.

The following screenshot shows the output of the regression model:

Output of simple linear regression in Excel

The fitted simple linear regression model is:

Exam Score = 67.1617 + 5.2503*(hours studied)

To determine if there is a statistically significant relationship between hours studied and exam score, we need to analyze the overall F value of the model and the corresponding p-value:

  • Overall F-Value:  47.9952
  • P-value:  0.000

Since this p-value is less than .05, we can reject the null hypothesis. In other words, there is a statistically significant relationship between hours studied and exam score received.

Example 2: Multiple Linear Regression

Suppose a professor would like to use the number of hours studied and the number of prep exams taken to predict the exam score that students will receive in his class. He collects data for 20 students and fits a multiple linear regression model.

Multiple linear regression output in Excel

The fitted multiple linear regression model is:

Exam Score = 67.67 + 5.56*(hours studied) – 0.60*(prep exams taken)

To determine if there is a jointly statistically significant relationship between the two predictor variables and the response variable, we need to analyze the overall F value of the model and the corresponding p-value:

  • Overall F-Value:  23.46
  • P-value:  0.00

Since this p-value is less than .05, we can reject the null hypothesis. In other words, hours studied and prep exams taken have a jointly statistically significant relationship with exam score.

Note: Although the p-value for prep exams taken (p = 0.52) is not significant, prep exams combined with hours studied has a significant relationship with exam score.

Additional Resources

Understanding the F-Test of Overall Significance in Regression How to Read and Interpret a Regression Table How to Report Regression Results How to Perform Simple Linear Regression in Excel How to Perform Multiple Linear Regression in Excel

Featured Posts

5 Statistical Biases to Avoid

Hey there. My name is Zach Bobbitt. I have a Master of Science degree in Applied Statistics and I’ve worked on machine learning algorithms for professional businesses in both healthcare and retail. I’m passionate about statistics, machine learning, and data visualization and I created Statology to be a resource for both students and teachers alike.  My goal with this site is to help you learn statistics through using simple terms, plenty of real-world examples, and helpful illustrations.

2 Replies to “Understanding the Null Hypothesis for Linear Regression”

Thank you Zach, this helped me on homework!

Great articles, Zach.

I would like to cite your work in a research paper.

Could you provide me with your last name and initials.

Leave a Reply Cancel reply

Your email address will not be published. Required fields are marked *

User Preferences

Content preview.

Arcu felis bibendum ut tristique et egestas quis:

  • Ut enim ad minim veniam, quis nostrud exercitation ullamco laboris
  • Duis aute irure dolor in reprehenderit in voluptate
  • Excepteur sint occaecat cupidatat non proident

Keyboard Shortcuts

6.4 - the hypothesis tests for the slopes.

At the beginning of this lesson, we translated three different research questions pertaining to heart attacks in rabbits ( Cool Hearts dataset ) into three sets of hypotheses we can test using the general linear F -statistic. The research questions and their corresponding hypotheses are:

Hypotheses 1

Is the regression model containing at least one predictor useful in predicting the size of the infarct?

  • \(H_{0} \colon \beta_{1} = \beta_{2} = \beta_{3} = 0\)
  • \(H_{A} \colon\) At least one \(\beta_{j} ≠ 0\) (for j = 1, 2, 3)

Hypotheses 2

Is the size of the infarct significantly (linearly) related to the area of the region at risk?

  • \(H_{0} \colon \beta_{1} = 0 \)
  • \(H_{A} \colon \beta_{1} \ne 0 \)

Hypotheses 3

(Primary research question) Is the size of the infarct area significantly (linearly) related to the type of treatment upon controlling for the size of the region at risk for infarction?

  • \(H_{0} \colon \beta_{2} = \beta_{3} = 0\)
  • \(H_{A} \colon \) At least one \(\beta_{j} ≠ 0\) (for j = 2, 3)

Let's test each of the hypotheses now using the general linear F -statistic:

\(F^*=\left(\dfrac{SSE(R)-SSE(F)}{df_R-df_F}\right) \div \left(\dfrac{SSE(F)}{df_F}\right)\)

To calculate the F -statistic for each test, we first determine the error sum of squares for the reduced and full models — SSE ( R ) and SSE ( F ), respectively. The number of error degrees of freedom associated with the reduced and full models — \(df_{R}\) and \(df_{F}\), respectively — is the number of observations, n , minus the number of parameters, p , in the model. That is, in general, the number of error degrees of freedom is n - p . We use statistical software, such as Minitab's F -distribution probability calculator, to determine the P -value for each test.

Testing all slope parameters equal 0 Section  

To answer the research question: "Is the regression model containing at least one predictor useful in predicting the size of the infarct?" To do so, we test the hypotheses:

  • \(H_{0} \colon \beta_{1} = \beta_{2} = \beta_{3} = 0 \)
  • \(H_{A} \colon\) At least one \(\beta_{j} \ne 0 \) (for j = 1, 2, 3)

The full model

The full model is the largest possible model — that is, the model containing all of the possible predictors. In this case, the full model is:

\(y_i=(\beta_0+\beta_1x_{i1}+\beta_2x_{i2}+\beta_3x_{i3})+\epsilon_i\)

The error sum of squares for the full model, SSE ( F ), is just the usual error sum of squares, SSE , that appears in the analysis of variance table. Because there are 4 parameters in the full model, the number of error degrees of freedom associated with the full model is \(df_{F} = n - 4\).

The reduced model

The reduced model is the model that the null hypothesis describes. Because the null hypothesis sets each of the slope parameters in the full model equal to 0, the reduced model is:

\(y_i=\beta_0+\epsilon_i\)

The reduced model suggests that none of the variations in the response y is explained by any of the predictors. Therefore, the error sum of squares for the reduced model, SSE ( R ), is just the total sum of squares, SSTO , that appears in the analysis of variance table. Because there is only one parameter in the reduced model, the number of error degrees of freedom associated with the reduced model is \(df_{R} = n - 1 \).

Upon plugging in the above quantities, the general linear F -statistic:

\(F^*=\dfrac{SSE(R)-SSE(F)}{df_R-df_F} \div \dfrac{SSE(F)}{df_F}\)

becomes the usual " overall F -test ":

\(F^*=\dfrac{SSR}{3} \div \dfrac{SSE}{n-4}=\dfrac{MSR}{MSE}\)

That is, to test \(H_{0}\) : \(\beta_{1} = \beta_{2} = \beta_{3} = 0 \), we just use the overall F -test and P -value reported in the analysis of variance table:

Analysis of Variance

Regression equation.

Inf = - 0.135 + 0.613 Area - 0.2435 X2 - 0.0657 X3

There is sufficient evidence ( F = 16.43, P < 0.001) to conclude that at least one of the slope parameters is not equal to 0.

In general, to test that all of the slope parameters in a multiple linear regression model are 0, we use the overall F -test reported in the analysis of variance table.

Testing one slope parameter is 0 Section  

Now let's answer the second research question: "Is the size of the infarct significantly (linearly) related to the area of the region at risk?" To do so, we test the hypotheses:

Again, the full model is the model containing all of the possible predictors:

The error sum of squares for the full model, SSE ( F ), is just the usual error sum of squares, SSE . Alternatively, because the three predictors in the model are \(x_{1}\), \(x_{2}\), and \(x_{3}\), we can denote the error sum of squares as SSE (\(x_{1}\), \(x_{2}\), \(x_{3}\)). Again, because there are 4 parameters in the model, the number of error degrees of freedom associated with the full model is \(df_{F} = n - 4 \).

Because the null hypothesis sets the first slope parameter, \(\beta_{1}\), equal to 0, the reduced model is:

\(y_i=(\beta_0+\beta_2x_{i2}+\beta_3x_{i3})+\epsilon_i\)

Because the two predictors in the model are \(x_{2}\) and \(x_{3}\), we denote the error sum of squares as SSE (\(x_{2}\), \(x_{3}\)). Because there are 3 parameters in the model, the number of error degrees of freedom associated with the reduced model is \(df_{R} = n - 3\).

The general linear statistic:

simplifies to:

\(F^*=\dfrac{SSR(x_1|x_2, x_3)}{1}\div \dfrac{SSE(x_1,x_2, x_3)}{n-4}=\dfrac{MSR(x_1|x_2, x_3)}{MSE(x_1,x_2, x_3)}\)

Getting the numbers from the Minitab output:

we determine that the value of the F -statistic is:

\(F^* = \dfrac{SSR(x_1 \vert x_2, x_3)}{1} \div \dfrac{SSE(x_1, x_2, x_3)}{28} = \dfrac{0.63742}{0.01946}=32.7554\)

The P -value is the probability — if the null hypothesis were true — that we would get an F -statistic larger than 32.7554. Comparing our F -statistic to an F -distribution with 1 numerator degree of freedom and 28 denominator degrees of freedom, Minitab tells us that the probability is close to 1 that we would observe an F -statistic smaller than 32.7554:

F distribution with 1 DF in Numerator and 28 DF in denominator

Therefore, the probability that we would get an F -statistic larger than 32.7554 is close to 0. That is, the P -value is < 0.001. There is sufficient evidence ( F = 32.8, P < 0.001) to conclude that the size of the infarct is significantly related to the size of the area at risk after the other predictors x2 and x3 have been taken into account.

But wait a second! Have you been wondering why we couldn't just use the slope's t -statistic to test that the slope parameter, \(\beta_{1}\), is 0? We can! Notice that the P -value ( P < 0.001) for the t -test ( t * = 5.72):

Coefficients

is the same as the P -value we obtained for the F -test. This will always be the case when we test that only one slope parameter is 0. That's because of the well-known relationship between a t -statistic and an F -statistic that has one numerator degree of freedom:

\(t_{(n-p)}^{2}=F_{(1, n-p)}\)

For our example, the square of the t -statistic, 5.72, equals our F -statistic (within rounding error). That is:

\(t^{*2}=5.72^2=32.72=F^*\)

So what have we learned in all of this discussion about the equivalence of the F -test and the t -test? In short:

Compare the output obtained when \(x_{1}\) = Area is entered into the model last :

Inf = - 0.135 - 0.2435 X2 - 0.0657 X3 + 0.613 Area

to the output obtained when \(x_{1}\) = Area is entered into the model first :

The t -statistic and P -value are the same regardless of the order in which \(x_{1}\) = Area is entered into the model. That's because — by its equivalence to the F -test — the t -test for one slope parameter adjusts for all of the other predictors included in the model.

  • We can use either the F -test or the t -test to test that only one slope parameter is 0. Because the t -test results can be read right off of the Minitab output, it makes sense that it would be the test that we'll use most often.
  • But, we have to be careful with our interpretations! The equivalence of the t -test to the F -test has taught us something new about the t -test. The t -test is a test for the marginal significance of the \(x_{1}\) predictor after the other predictors \(x_{2}\) and \(x_{3}\) have been taken into account. It does not test for the significance of the relationship between the response y and the predictor \(x_{1}\) alone.

Testing a subset of slope parameters is 0 Section  

Finally, let's answer the third — and primary — research question: "Is the size of the infarct area significantly (linearly) related to the type of treatment upon controlling for the size of the region at risk for infarction?" To do so, we test the hypotheses:

  • \(H_{0} \colon \beta_{2} = \beta_{3} = 0 \)
  • \(H_{A} \colon\) At least one \(\beta_{j} \ne 0 \) (for j = 2, 3)

Because the null hypothesis sets the second and third slope parameters, \(\beta_{2}\) and \(\beta_{3}\), equal to 0, the reduced model is:

\(y_i=(\beta_0+\beta_1x_{i1})+\epsilon_i\)

The ANOVA table for the reduced model is:

Because the only predictor in the model is \(x_{1}\), we denote the error sum of squares as SSE (\(x_{1}\)) = 0.8793. Because there are 2 parameters in the model, the number of error degrees of freedom associated with the reduced model is \(df_{R} = n - 2 = 32 – 2 = 30\).

\begin{align} F^*&=\dfrac{SSE(R)-SSE(F)}{df_R-df_F} \div\dfrac{SSE(F)}{df_F}\\&=\dfrac{0.8793-0.54491}{30-28} \div\dfrac{0.54491}{28}\\&= \dfrac{0.33439}{2} \div 0.01946\\&=8.59.\end{align}

Alternatively, we can calculate the F-statistic using a partial F-test :

\begin{align}F^*&=\dfrac{SSR(x_2, x_3|x_1)}{2}\div \dfrac{SSE(x_1,x_2, x_3)}{n-4}\\&=\dfrac{MSR(x_2, x_3|x_1)}{MSE(x_1,x_2, x_3)}.\end{align}

To conduct the test, we regress y = InfSize on \(x_{1}\) = Area and \(x_{2}\) and \(x_{3 }\)— in order (and with "Sequential sums of squares" selected under "Options"):

Inf = - 0.135 + 0.613 Area - 0.2435 X2 - 0.0657 X3

yielding SSR (\(x_{2}\) | \(x_{1}\)) = 0.31453, SSR (\(x_{3}\) | \(x_{1}\), \(x_{2}\)) = 0.01981, and MSE = 0.54491/28 = 0.01946. Therefore, the value of the partial F -statistic is:

\begin{align} F^*&=\dfrac{SSR(x_2, x_3|x_1)}{2}\div \dfrac{SSE(x_1,x_2, x_3)}{n-4}\\&=\dfrac{0.31453+0.01981}{2}\div\dfrac{0.54491}{28}\\&= \dfrac{0.33434}{2} \div 0.01946\\&=8.59,\end{align}

which is identical (within round-off error) to the general F-statistic above. The P -value is the probability — if the null hypothesis were true — that we would observe a partial F -statistic more extreme than 8.59. The following Minitab output:

F distribution with 2 DF in Numerator and 28 DF in denominator

tells us that the probability of observing such an F -statistic that is smaller than 8.59 is 0.9988. Therefore, the probability of observing such an F -statistic that is larger than 8.59 is 1 - 0.9988 = 0.0012. The P -value is very small. There is sufficient evidence ( F = 8.59, P = 0.0012) to conclude that the type of cooling is significantly related to the extent of damage that occurs — after taking into account the size of the region at risk.

Summary of MLR Testing Section  

For the simple linear regression model, there is only one slope parameter about which one can perform hypothesis tests. For the multiple linear regression model, there are three different hypothesis tests for slopes that one could conduct. They are:

  • Hypothesis test for testing that all of the slope parameters are 0.
  • Hypothesis test for testing that a subset — more than one, but not all — of the slope parameters are 0.
  • Hypothesis test for testing that one slope parameter is 0.

We have learned how to perform each of the above three hypothesis tests. Along the way, we also took two detours — one to learn about the " general linear F-test " and one to learn about " sequential sums of squares. " As you now know, knowledge about both is necessary for performing the three hypothesis tests.

The F -statistic and associated p -value in the ANOVA table is used for testing whether all of the slope parameters are 0. In most applications, this p -value will be small enough to reject the null hypothesis and conclude that at least one predictor is useful in the model. For example, for the rabbit heart attacks study, the F -statistic is (0.95927/(4–1)) / (0.54491/(32–4)) = 16.43 with p -value 0.000.

To test whether a subset — more than one, but not all — of the slope parameters are 0, there are two equivalent ways to calculate the F-statistic:

  • Use the general linear F-test formula by fitting the full model to find SSE(F) and fitting the reduced model to find SSE(R) . Then the numerator of the F-statistic is (SSE(R) – SSE(F)) / ( \(df_{R}\) – \(df_{F}\)) .
  • Alternatively, use the partial F-test formula by fitting only the full model but making sure the relevant predictors are fitted last and "sequential sums of squares" have been selected. Then the numerator of the F-statistic is the sum of the relevant sequential sums of squares divided by the sum of the degrees of freedom for these sequential sums of squares. The denominator of the F -statistic is the mean squared error in the ANOVA table.

For example, for the rabbit heart attacks study, the general linear F-statistic is ((0.8793 – 0.54491) / (30 – 28)) / (0.54491 / 28) = 8.59 with p -value 0.0012. Alternatively, the partial F -statistic for testing the slope parameters for predictors \(x_{2}\) and \(x_{3}\) using sequential sums of squares is ((0.31453 + 0.01981) / 2) / (0.54491 / 28) = 8.59.

To test whether one slope parameter is 0, we can use an F -test as just described. Alternatively, we can use a t -test, which will have an identical p -value since in this case, the square of the t -statistic is equal to the F -statistic. For example, for the rabbit heart attacks study, the F -statistic for testing the slope parameter for the Area predictor is (0.63742/1) / (0.54491/(32–4)) = 32.75 with p -value 0.000. Alternatively, the t -statistic for testing the slope parameter for the Area predictor is 0.613 / 0.107 = 5.72 with p -value 0.000, and \(5.72^{2} = 32.72\).

Incidentally, you may be wondering why we can't just do a series of individual t-tests to test whether a subset of the slope parameters is 0. For example, for the rabbit heart attacks study, we could have done the following:

  • Fit the model of y = InfSize on \(x_{1}\) = Area and \(x_{2}\) and \(x_{3}\) and use an individual t-test for \(x_{3}\).
  • If the test results indicate that we can drop \(x_{3}\) then fit the model of y = InfSize on \(x_{1}\) = Area and \(x_{2}\) and use an individual t-test for \(x_{2}\).

The problem with this approach is we're using two individual t-tests instead of one F-test, which means our chance of drawing an incorrect conclusion in our testing procedure is higher. Every time we do a hypothesis test, we can draw an incorrect conclusion by:

  • rejecting a true null hypothesis, i.e., make a type I error by concluding the tested predictor(s) should be retained in the model when in truth it/they should be dropped; or
  • failing to reject a false null hypothesis, i.e., make a type II error by concluding the tested predictor(s) should be dropped from the model when in truth it/they should be retained.

Thus, in general, the fewer tests we perform the better. In this case, this means that wherever possible using one F-test in place of multiple individual t-tests is preferable.

Hypothesis tests for the slope parameters Section  

The problems in this section are designed to review the hypothesis tests for the slope parameters, as well as to give you some practice on models with a three-group qualitative variable (which we'll cover in more detail in Lesson 8). We consider tests for:

  • whether one slope parameter is 0 (for example, \(H_{0} \colon \beta_{1} = 0 \))
  • whether a subset (more than one but less than all) of the slope parameters are 0 (for example, \(H_{0} \colon \beta_{2} = \beta_{3} = 0 \) against the alternative \(H_{A} \colon \beta_{2} \ne 0 \) or \(\beta_{3} \ne 0 \) or both ≠ 0)
  • whether all of the slope parameters are 0 (for example, \(H_{0} \colon \beta_{1} = \beta_{2} = \beta_{3}\) = 0 against the alternative \(H_{A} \colon \) at least one of the \(\beta_{i}\) is not 0)

(Note the correct specification of the alternative hypotheses for the last two situations.)

Sugar beets study

A group of researchers was interested in studying the effects of three different growth regulators ( treat , denoted 1, 2, and 3) on the yield of sugar beets (y = yield , in pounds). They planned to plant the beets in 30 different plots and then randomly treat 10 plots with the first growth regulator, 10 plots with the second growth regulator, and 10 plots with the third growth regulator. One problem, though, is that the amount of available nitrogen in the 30 different plots varies naturally, thereby giving a potentially unfair advantage to plots with higher levels of available nitrogen. Therefore, the researchers also measured and recorded the available nitrogen (\(x_{1}\) = nit , in pounds/acre) in each plot. They are interested in comparing the mean yields of sugar beets subjected to the different growth regulators after taking into account the available nitrogen. The Sugar Beets dataset contains the data from the researcher's experiment.

Preliminary Work

The plot shows a similar positive linear trend within each treatment category, which suggests that it is reasonable to formulate a multiple regression model that would place three parallel lines through the data.

Because the qualitative variable treat distinguishes between the three treatment groups (1, 2, and 3), we need to create two indicator variables, \(x_{2}\) and \(x_{3}\), say, to fit a linear regression model to these data. The new indicator variables should be defined as follows:

Use Minitab's Calc >> Make Indicator Variables command to create the new indicator variables in your worksheet

Minitab creates an indicator variable for each treatment group but we can only use two, for treatment groups 1 and 2 in this case (treatment group 3 is the reference level in this case).

Then, if we assume the trend in the data can be summarized by this regression model:

\(y_{i} = \beta_{0}\) + \(\beta_{1}\)\(x_{1}\) + \(\beta_{2}\)\(x_{2}\) + \(\beta_{3}\)\(x_{3}\) + \(\epsilon_{i}\)

where \(x_{1}\) = nit and \(x_{2}\) and \(x_{3}\) are defined as above, what is the mean response function for plots receiving treatment 3? for plots receiving treatment 1? for plots receiving treatment 2? Are the three regression lines that arise from our formulated model parallel? What does the parameter \(\beta_{2}\) quantify? And, what does the parameter \(\beta_{3}\) quantify?

The fitted equation from Minitab is Yield = 84.99 + 1.3088 Nit - 2.43 \(x_{2}\) - 2.35 \(x_{3}\), which means that the equations for each treatment group are:

  • Group 1: Yield = 84.99 + 1.3088 Nit - 2.43(1) = 82.56 + 1.3088 Nit
  • Group 2: Yield = 84.99 + 1.3088 Nit - 2.35(1) = 82.64 + 1.3088 Nit
  • Group 3: Yield = 84.99 + 1.3088 Nit

The three estimated regression lines are parallel since they have the same slope, 1.3088.

The regression parameter for \(x_{2}\) represents the difference between the estimated intercept for treatment 1 and the estimated intercept for reference treatment 3.

The regression parameter for \(x_{3}\) represents the difference between the estimated intercept for treatment 2 and the estimated intercept for reference treatment 3.

Testing whether all of the slope parameters are 0

\(H_0 \colon \beta_1 = \beta_2 = \beta_3 = 0\) against the alternative \(H_A \colon \) at least one of the \(\beta_i\) is not 0.

\(F=\dfrac{SSR(X_1,X_2,X_3)\div3}{SSE(X_1,X_2,X_3)\div(n-4)}=\dfrac{MSR(X_1,X_2,X_3)}{MSE(X_1,X_2,X_3)}\)

\(F = \dfrac{\frac{16039.5}{3}}{\frac{1078.0}{30-4}} = \dfrac{5346.5}{41.46} = 128.95\)

Since the p -value for this F -statistic is reported as 0.000, we reject \(H_{0}\) in favor of \(H_{A}\) and conclude that at least one of the slope parameters is not zero, i.e., the regression model containing at least one predictor is useful in predicting the size of sugar beet yield.

Tests for whether one slope parameter is 0

\(H_0 \colon \beta_1= 0\) against the alternative \(H_A \colon \beta_1 \ne 0\)

t -statistic = 19.60, p -value = 0.000, so we reject \(H_{0}\) in favor of \(H_{A}\) and conclude that the slope parameter for \(x_{1}\) = nit is not zero, i.e., sugar beet yield is significantly linearly related to the available nitrogen (controlling for treatment).

\(F=\dfrac{SSR(X_1|X_2,X_3)\div1}{SSE(X_1,X_2,X_3)\div(n-4)}=\dfrac{MSR(X_1|X_2,X_3)}{MSE(X_1,X_2,X_3)}\)

Use the Minitab output to calculate the value of this F statistic. Does the value you obtain equal \(t^{2}\), the square of the t -statistic as we might expect?

\(F-statistic= \dfrac{\frac{15934.5}{1}}{\frac{1078.0}{30-4}} = \dfrac{15934.5}{41.46} = 384.32\), which is the same as \(19.60^{2}\).

Because \(t^{2}\) will equal the partial F -statistic whenever you test for whether one slope parameter is 0, it makes sense to just use the t -statistic and P -value that Minitab displays as a default. But, note that we've just learned something new about the meaning of the t -test in the multiple regression setting. It tests for the ("marginal") significance of the \(x_{1}\) predictor after \(x_{2}\) and \(x_{3}\) have already been taken into account.

Tests for whether a subset of the slope parameters is 0

\(H_0 \colon \beta_2=\beta_3= 0\) against the alternative \(H_A \colon \beta_2 \ne 0\) or \(\beta_3 \ne 0\) or both \(\ne 0\).

\(F=\dfrac{SSR(X_2,X_3|X_1)\div2}{SSE(X_1,X_2,X_3)\div(n-4)}=\dfrac{MSR(X_2,X_3|X_1)}{MSE(X_1,X_2,X_3)}\)

\(F = \dfrac{\frac{10.4+27.5}{2}}{\frac{1078.0}{30-4}} = \dfrac{18.95}{41.46} = 0.46\).

F distribution with 2 DF in Numerator and 26 DF in denominator

p-value \(= 1-0.363677 = 0.636\), so we fail to reject \(H_{0}\) in favor of \(H_{A}\) and conclude that we cannot rule out \(\beta_2 = \beta_3 = 0\), i.e., there is no significant difference in the mean yields of sugar beets subjected to the different growth regulators after taking into account the available nitrogen.

Note that the sequential mean square due to regression, MSR(\(X_{2}\),\(X_{3}\)|\(X_{1}\)), is obtained by dividing the sequential sum of square by its degrees of freedom (2, in this case, since two additional predictors \(X_{2}\) and \(X_{3}\) are considered). Use the Minitab output to calculate the value of this F statistic, and use Minitab to get the associated P -value. Answer the researcher's question at the \(\alpha= 0.05\) level.

IMAGES

  1. Hypothesis Test in Multiple Linear Regression, Part 1

    hypothesis test for multiple linear regression

  2. PPT

    hypothesis test for multiple linear regression

  3. Multiple Linear Regression Hypothesis Testing in Matrix Form

    hypothesis test for multiple linear regression

  4. Mod-01 Lec-39 Hypothesis Testing in Linear Regression

    hypothesis test for multiple linear regression

  5. PPT

    hypothesis test for multiple linear regression

  6. Multiple Linear Regression

    hypothesis test for multiple linear regression

VIDEO

  1. Hypothesis Testing in Simple Linear Regression

  2. Simple linear regression hypothesis testing

  3. Regression and test of hypothesis

  4. Conduct a Multiple Linear Correlation Hypothesis Test Using Free Web Calculators

  5. Linear Regression

  6. اختبارات الفروض : تحليل الانحدار المتعدد Hypothesis tests: multiple regression analysis

COMMENTS

  1. PDF Lecture 5 Hypothesis Testing in Multiple Linear Regression

    As in simple linear regression, under the null hypothesis t 0 = βˆ j seˆ(βˆ j) ∼ t n−p−1. We reject H 0 if |t 0| > t n−p−1,1−α/2. This is a partial test because βˆ j depends on all of the other predictors x i, i 6= j that are in the model. Thus, this is a test of the contribution of x j given the other predictors in the model.

  2. Hypothesis Tests and Confidence Intervals in Multiple Regression

    Confidence Intervals for a Single Coefficient. The confidence interval for a regression coefficient in multiple regression is calculated and interpreted the same way as it is in simple linear regression. The t-statistic has n - k - 1 degrees of freedom where k = number of independents. Supposing that an interval contains the true value of ...

  3. Multiple Linear Regression

    The formula for a multiple linear regression is: = the predicted value of the dependent variable. = the y-intercept (value of y when all other parameters are set to 0) = the regression coefficient () of the first independent variable () (a.k.a. the effect that increasing the value of the independent variable has on the predicted y value ...

  4. Lesson 5: Multiple Linear Regression (MLR) Model & Evaluation

    a hypothesis test for testing that a subset — more than one, but not all — of the slope parameters are 0. In this lesson, we also learn how to perform each of the above three hypothesis tests. Key Learning Goals for this Lesson: Be able to interpret the coefficients of a multiple regression model. Understand what the scope of the model is ...

  5. 5.3

    A population model for a multiple linear regression model that relates a y -variable to p -1 x -variables is written as. y i = β 0 + β 1 x i, 1 + β 2 x i, 2 + … + β p − 1 x i, p − 1 + ϵ i. We assume that the ϵ i have a normal distribution with mean 0 and constant variance σ 2. These are the same assumptions that we used in simple ...

  6. PDF Hypothesis Testing in the Multiple regression model

    Testing that individual coefficients take a specific value such as zero or some other value is done in exactly the same way as with the simple two variable regression model. Now suppose we wish to test that a number of coefficients or combinations of coefficients take some particular value. In this case we will use the so called "F-test".

  7. 15.5: Hypothesis Tests for Regression Models

    Okay, suppose you've estimated your regression model. The first hypothesis test you might want to try is one in which the null hypothesis that there is no relationship between the predictors and the outcome, ... 4.354 on 97 degrees of freedom Multiple R-squared: 0.8161, Adjusted R-squared: 0.8123 F-statistic: 215.2 on 2 and 97 DF, p-value ...

  8. 9.5

    F-Test for Overall Significance. There is a statistical test we can use to determine the overall significance of the regression model. The F-test in Multiple Linear Regression test the following hypotheses: H 0: β 1 =... = β k = 0. H a: At least one β i is not equal to zero. The test statistic for this test, denoted F ∗, follows an F ...

  9. Multiple Regression: Estimation and Hypothesis Testing

    Multiple Regression: Estimation and Hypothesis Testing. ... Although in many ways a straightforward extension of the two-variable linear regression model, the three-variable model introduced several new concepts, such as partial regression coefficients, adjusted and unadjusted multiple coefficient of determination, and multicollinearity ...

  10. Multiple linear regression

    However, hypothesis tests derived from these variables are affected by the choice. Solution: To check whether region is important, use an \ ... Defined Multiple Linear Regression. Discussed how to test the importance of variables. Described one approach to choose a subset of variables.

  11. Hypothesis Tests in Multiple Linear Regression, Part 1

    Organized by textbook: https://learncheme.com/ See Part 2: https://www.youtube.com/watch?v=ziGbG0dRlsAMade by faculty at the University of Colorado Boulder, ...

  12. 12.2.1: Hypothesis Test for Linear Regression

    The hypotheses are: Find the critical value using dfE = n − p − 1 = 13 for a two-tailed test α = 0.05 inverse t-distribution to get the critical values ± 2.160. Draw the sampling distribution and label the critical values, as shown in Figure 12-14. Figure 12-14: Graph of t-distribution with labeled critical values.

  13. Hypothesis Tests in Multiple Regression

    In this lecture video we test hypotheses about the beta parameters in the multiple linear regression model.Next lecture in this series: https://youtu.be/rj1O...

  14. Multiple linear regression -- Advanced Statistics using R

    Hypothesis testing of regression coefficient(s) With the estimates of regression coefficients and their standard errors estimates, we can conduct hypothesis testing for one, a subset, or all regression coefficients. ... about 40% of the variation in college GPA can be explained by the multiple linear regression with h.GPA, SAT, and recommd as ...

  15. Lesson 5: Multiple Linear Regression

    Minitab Help 5: Multiple Linear Regression; R Help 5: Multiple Linear Regression; Lesson 6: MLR Model Evaluation. 6.1 - Three Types of Hypotheses; 6.2 - The General Linear F-Test; 6.3 - Sequential (or Extra) Sums of Squares; 6.4 - The Hypothesis Tests for the Slopes; 6.5 - Partial R-squared; 6.6 - Lack of Fit Testing in the Multiple Regression ...

  16. Multiple Linear Regression. A complete study

    Alright! We understood Linear Regression, we built the model and even interpreted the results. What we learned so far were the fundamentals of Linear Regression. However, while dealing with real-world problems, we generally go beyond this point to statistically analyze our model and do the necessary changes if required. Hypothesis Test for ...

  17. 8.7: Overall F-test in multiple linear regression

    This test is called the overall F-test in MLR and is very similar to the F F -test in a reference-coded One-Way ANOVA model. It tests the null hypothesis that involves setting every coefficient except the y y -intercept to 0 (so all the slope coefficients equal 0). We saw this reduced model in the One-Way material when we considered setting all ...

  18. Multiple Linear Regression

    Multiple linear regression analysis is the creation of an equation with multiple independent X variables that all influence a Y response variable. This equation is based upon an existing data set and models the conditions represented in the data. ... 01:09 the hypothesis tests are still the same as a simple linear regression. 01:13 The null ...

  19. Writing hypothesis for linear multiple regression models

    2. I struggle writing hypothesis because I get very much confused by reference groups in the context of regression models. For my example I'm using the mtcars dataset. The predictors are wt (weight), cyl (number of cylinders), and gear (number of gears), and the outcome variable is mpg (miles per gallon). Say all your friends think you should ...

  20. 5.7

    For the simple linear regression model, there is only one slope parameter about which one can perform hypothesis tests. For the multiple linear regression model, there are three different hypothesis tests for slopes that one could conduct. They are: Hypothesis test for testing that all of the slope parameters are 0. Hypothesis test for testing ...

  21. Multiple Linear Regression in SPSS

    The hypothesis testing in Multiple Linear Regression revolves around assessing whether the collective set of independent variables has a statistically significant impact on the dependent variable. The null hypothesis suggests no overall effect, while the alternative hypothesis asserts the presence of at least one significant relationship.

  22. Understanding the Null Hypothesis for Linear Regression

    xi: The value of the predictor variable xi. Multiple linear regression uses the following null and alternative hypotheses: H0: β1 = β2 = … = βk = 0. HA: β1 = β2 = … = βk ≠ 0. The null hypothesis states that all coefficients in the model are equal to zero. In other words, none of the predictor variables have a statistically ...

  23. 6.4

    For the simple linear regression model, there is only one slope parameter about which one can perform hypothesis tests. For the multiple linear regression model, there are three different hypothesis tests for slopes that one could conduct. They are: Hypothesis test for testing that all of the slope parameters are 0.